Re: [Gretl-users] Holt-Winters package

2016-05-24 Thread Paolo Chirico
Dear Ignacio, I'm trying to estimate the parameters of a HW method by means of MLE command: mle ll = -1/2*log(2*$pi*s2)-1/2*(e^2)/s2 loop i=1+s...$nobs (HW method) endloop params alpha beta gamma end mle Unfortunately the loop command is not permitted by mle commnd! Have you any idea/advice to

Re: [Gretl-users] Holt-Winters package

2015-10-01 Thread Ignacio Diaz-Emparanza
El 30/09/15 a las 17:55, Raul Gimeno escribió: > Thank you Ignacio for your answer. > > Would it be possible to modify your program such that the initial values > correspond either to a fixed value given manually or correspond to the OLS > estimation of the parameters for the whole times series or

[Gretl-users] Holt-Winters package

2015-09-30 Thread Raul Gimeno
; >> ** >> >> >> ___ >> Gretl-users mailing list >> Gretl-users(a)lists.wfu.edu >> http://lists.wfu.edu/mailman/listinfo/gretl-users > > > > -- > Ignacio D?az-Emparanza > Departamento de Econom?a A

Re: [Gretl-users] Holt-Winters package

2015-09-29 Thread Ignacio Diaz-Emparanza
El 28/09/15 a las 22:40, Pedro Bação escribió: > Some time ago I came across a similar problem when using this function > for a class. At the time I made a note to myself saying that the > problem was in the definition of lobs, which I changed to: >scalar lobs=lastobs(y) > I hope this helps >

Re: [Gretl-users] Holt-Winters package

2015-09-28 Thread Pedro Bação
Some time ago I came across a similar problem when using this function for a class. At the time I made a note to myself saying that the problem was in the definition of lobs, which I changed to: scalar lobs=lastobs(y) I hope this helps On 28 September 2015 at 15:56, Ignacio Diaz-Emparanza

Re: [Gretl-users] Holt-Winters package

2015-09-28 Thread Ignacio Diaz-Emparanza
I have not much time for testing today, but it seems that a correction I included to avoid initial conditions very different from the first observations of the series is not working well for your data. If you change in the HoltWinters package the line series yh1= (0.85*y<= $yhat && $yhat

[Gretl-users] Holt-Winters package

2015-09-28 Thread Raul Gimeno
Hello I've been using the Holt-Winters package but I cannot replicate my Excel-calculation results with this package. The starting value from the package for the trend is 245 mine is 166.396. By running a regression on the full sample I get completely different results for these starting values,